Difference between revisions of "2021 AIME I Problems/Problem 13"
Sugar rush (talk | contribs) |
Sugar rush (talk | contribs) |
||
Line 2: | Line 2: | ||
Circles <math>\omega_1</math> and <math>\omega_2</math> with radii <math>961</math> and <math>625</math>, respectively, intersect at distinct points <math>A</math> and <math>B</math>. A third circle <math>\omega</math> is externally tangent to both <math>\omega_1</math> and <math>\omega_2</math>. Suppose line <math>AB</math> intersects <math>\omega</math> at two points <math>P</math> and <math>Q</math> such that the measure of minor arc <math>\widehat{PQ}</math> is <math>120^{\circ}</math>. What is the distance between the centers of <math>\omega_1</math> and <math>\omega_2</math>? | Circles <math>\omega_1</math> and <math>\omega_2</math> with radii <math>961</math> and <math>625</math>, respectively, intersect at distinct points <math>A</math> and <math>B</math>. A third circle <math>\omega</math> is externally tangent to both <math>\omega_1</math> and <math>\omega_2</math>. Suppose line <math>AB</math> intersects <math>\omega</math> at two points <math>P</math> and <math>Q</math> such that the measure of minor arc <math>\widehat{PQ}</math> is <math>120^{\circ}</math>. What is the distance between the centers of <math>\omega_1</math> and <math>\omega_2</math>? | ||
− | ==Solution== | + | ==Solution by pad== |
− | <math>672</math> | + | Let <math>O_i</math> and <math>r_i</math> be the center and radius <math>\omega_i</math>, and let <math>O</math> and <math>r</math> be the center and radius of <math>\omega</math>. |
+ | |||
+ | Since <math>\overline{AB}</math> extends to an arc with arc <math>120^\circ</math>, the distance from <math>O</math> to <math>\overline{AB}</math> is <math>r/2</math>. Let <math>X=\overline{AB}\cap \overline{O_1O_2}</math>. Consider <math>\triangle OO_1O_2</math>. The line <math>\overline{AB}</math> is perpendicular to <math>\overline{O_1O_2}</math> and passes through <math>X</math>. Let <math>H</math> be the foot from <math>O</math> to <math>\overline{O_1O_2}</math>; so <math>HX=r/2</math>. We have by tangency <math>OO_1=r+r_1</math> and <math>OO_2=r+r_2</math>. Let <math>O_1O_2=d</math>. | ||
+ | [asy] | ||
+ | unitsize(3cm); | ||
+ | pointpen=black; pointfontpen=fontsize(9); | ||
+ | |||
+ | pair A=dir(110), B=dir(230), C=dir(310); | ||
+ | |||
+ | |||
+ | DPA(A--B--C--A); | ||
+ | |||
+ | |||
+ | |||
+ | pair H = foot(A, B, C); | ||
+ | draw(A--H); | ||
+ | pair X = 0.3*B + 0.7*C; | ||
+ | pair Y = A+X-H; | ||
+ | draw(X--1.3*Y-0.3*X); | ||
+ | draw(A--Y, dotted); | ||
+ | |||
+ | pair R1 = 1.3*X-0.3*Y; | ||
+ | pair R2 = 0.7*X+0.3*Y; | ||
+ | draw(R1--X); | ||
+ | |||
+ | |||
+ | D("O",A,dir(A)); | ||
+ | D("O_1",B,dir(B)); | ||
+ | D("O_2",C,dir(C)); | ||
+ | D("H",H,dir(270)); | ||
+ | D("X",X,dir(225)); | ||
+ | D("A",R1,dir(180)); | ||
+ | D("B",R2,dir(180)); | ||
+ | |||
+ | draw(rightanglemark(Y,X,C,3)); | ||
+ | |||
+ | |||
+ | [/asy] | ||
+ | Since <math>X</math> is on the radical axis of <math>\omega_1</math> and <math>\omega_2</math>, it has equal power, so | ||
+ | <cmath> O_1X^2 - r_1^2 = O_2X^2-r_2^2 \implies O_1X-O_2X = \frac{r_1^2-r_2^2}{d} </cmath>since <math>O_1X+O_2X=d</math>. Now we can solve for <math>O_1X</math> and <math>O_2X</math>, and in particular, | ||
+ | \begin{align*} | ||
+ | O_1H &= O_1X - HX = \frac{d+\frac{r_1^2-r_2^2}{d}}{2} - \frac{r}{2} \\ | ||
+ | O_2H &= O_2X + HX = \frac{d-\frac{r_1^2-r_2^2}{d}}{2} + \frac{r}{2}. | ||
+ | \end{align*}We want to solve for <math>d</math>. By the Pythagorean Theorem (twice): | ||
+ | \begin{align*} | ||
+ | &\qquad OH^2 = O_2H^2 - (r+r_2)^2 = O_1H^2 - (r+r_1)^2 \\ | ||
+ | &\implies \left(d+r-\tfrac{r_1^2-r_2^2}{d}\right)^2 - 4(r+r_2)^2 = \left(d-r+\tfrac{r_1^2-r_2^2}{d}\right)^2 - 4(r+r_1)^2 \\ | ||
+ | &\implies 2dr - 2(r_1^2-r_2)^2-8rr_2-4r_2^2 = -2dr+2(r_1^2-r_2^2)-8rr_1-4r_1^2 \\ | ||
+ | &\implies 4dr = 8rr_2-8rr_1 \\ | ||
+ | &\implies \boxed{d=2r_2-2r_1}. | ||
+ | \end{align*}Therefore, <math>d=2(r_2-r_1) = 2(961-625)=\boxed{672}</math>. | ||
==See also== | ==See also== | ||
{{AIME box|year=2021|n=I|num-b=12|num-a=14}} | {{AIME box|year=2021|n=I|num-b=12|num-a=14}} | ||
+ | |||
+ | [[Category:Intermediate Geometry Problems]] | ||
{{MAA Notice}} | {{MAA Notice}} |
Revision as of 15:48, 11 March 2021
Problem
Circles and with radii and , respectively, intersect at distinct points and . A third circle is externally tangent to both and . Suppose line intersects at two points and such that the measure of minor arc is . What is the distance between the centers of and ?
Solution by pad
Let and be the center and radius , and let and be the center and radius of .
Since extends to an arc with arc , the distance from to is . Let . Consider . The line is perpendicular to and passes through . Let be the foot from to ; so . We have by tangency and . Let . [asy] unitsize(3cm); pointpen=black; pointfontpen=fontsize(9);
pair A=dir(110), B=dir(230), C=dir(310);
DPA(A--B--C--A);
pair H = foot(A, B, C); draw(A--H); pair X = 0.3*B + 0.7*C; pair Y = A+X-H; draw(X--1.3*Y-0.3*X); draw(A--Y, dotted);
pair R1 = 1.3*X-0.3*Y; pair R2 = 0.7*X+0.3*Y; draw(R1--X);
D("O",A,dir(A));
D("O_1",B,dir(B));
D("O_2",C,dir(C));
D("H",H,dir(270));
D("X",X,dir(225));
D("A",R1,dir(180));
D("B",R2,dir(180));
draw(rightanglemark(Y,X,C,3));
[/asy]
Since is on the radical axis of and , it has equal power, so
since . Now we can solve for and , and in particular,
\begin{align*}
O_1H &= O_1X - HX = \frac{d+\frac{r_1^2-r_2^2}{d}}{2} - \frac{r}{2} \\
O_2H &= O_2X + HX = \frac{d-\frac{r_1^2-r_2^2}{d}}{2} + \frac{r}{2}.
\end{align*}We want to solve for . By the Pythagorean Theorem (twice):
\begin{align*}
&\qquad OH^2 = O_2H^2 - (r+r_2)^2 = O_1H^2 - (r+r_1)^2 \\
&\implies \left(d+r-\tfrac{r_1^2-r_2^2}{d}\right)^2 - 4(r+r_2)^2 = \left(d-r+\tfrac{r_1^2-r_2^2}{d}\right)^2 - 4(r+r_1)^2 \\
&\implies 2dr - 2(r_1^2-r_2)^2-8rr_2-4r_2^2 = -2dr+2(r_1^2-r_2^2)-8rr_1-4r_1^2 \\
&\implies 4dr = 8rr_2-8rr_1 \\
&\implies \boxed{d=2r_2-2r_1}.
\end{align*}Therefore, .
See also
2021 AIME I (Problems • Answer Key • Resources) | ||
Preceded by Problem 12 |
Followed by Problem 14 | |
1 • 2 • 3 • 4 • 5 • 6 • 7 • 8 • 9 • 10 • 11 • 12 • 13 • 14 • 15 | ||
All AIME Problems and Solutions |
The problems on this page are copyrighted by the Mathematical Association of America's American Mathematics Competitions.